In summary, the conversation discusses four real massive scalar fields, \phi_1,\phi_2,\phi_3, and \phi_4, coupled by the interaction lagrangian \mathcal{L}_{int}=\frac{-M_3}{2}\phi_1\phi_{3}^{2}-\frac{M_4}{2}\phi_2\phi_{4}^{2}. The goal is to find the scattering amplitude for \phi_{3}\phi_{4}\rightarrow\phi_3\phi_4, to tree level. The attempt at a solution involves considering the relevant Feynman diagrams and rules, but the Lagrangian suggests that the 13 and 24 sectors do
  • #1
MyName
10
0

Homework Statement


Consider four real massive scalar fields, [itex]\phi_1,\phi_2,\phi_3,[/itex] and [itex]\phi_4[/itex], with masses [itex]M_1,M_2,M_3,M_4[/itex].
Let these fields be coupled by the interaction lagrangian [itex]\mathcal{L}_{int}=\frac{-M_3}{2}\phi_1\phi_{3}^{2}-\frac{M_4}{2}\phi_2\phi_{4}^{2}[/itex].
Find the scattering amplitude for [itex]\phi_{3}\phi_{4}\rightarrow\phi_3\phi_4[/itex], to tree level.

Homework Equations


I'm not really sure what to put here.

The Attempt at a Solution


This honestly looks like a trick question to me. The first step is obviously to write down the relevant feynman diagrams and feynman rules to evaluate them, but I can't find a single tree level diagram for this process. A diagram would need an incoming [itex]\phi_3[/itex] and an incoming [itex]\phi_4[/itex], as well as an outgoing [itex]\phi_3[/itex] and an outgoing [itex]\phi_4[/itex].
The interaction lagrangian makes me think that we should have a vertex between a [itex]\phi_1[/itex] and two [itex]\phi_3[/itex]'s, as well as a vertex between a [itex]\phi_2[/itex] and two [itex]\phi_4[/itex]'s, but this still doesn't allow [itex]\phi_3[/itex] and [itex]\phi_4[/itex] to interact.
Perhaps I am misunderstanding the meaning of tree level (I think it just means one of each type of interaction vertex), or perhaps I am just misunderstanding the interactions.
Could this possibly have to do with interactions via the kinetic terms of the lagrangian? Can a tree level diagram just consist of the [itex]\phi_3[/itex] and the [itex]\phi_4[/itex] propagating along and not interacting (if yes I don't think this would contribute to the scattering), I'm not sure and would appreciate any help. I'm just feeling pretty confused at the moment.
Thanks!
 
Physics news on Phys.org
  • #2
With that Lagrangian the 13 and 24 sectors do not mix.
 
  • Like
Likes MyName
  • #3
Thanks, I thought that was the case and really appreciate the confirmation.
 

1. What are Feynman diagrams used for?

Feynman diagrams are graphical representations of mathematical equations used to describe the interactions between particles in quantum field theory. They are used to calculate the probability of different outcomes in particle interactions.

2. What are interacting scalar fields?

Interacting scalar fields are mathematical representations of particles that have no spin and do not interact with each other through the electromagnetic or strong nuclear force. They can interact through the Higgs field, which gives particles their mass.

3. How do Feynman diagrams help us understand particle interactions?

Feynman diagrams provide a visual representation of the mathematical equations that describe particle interactions. By analyzing these diagrams, scientists can better understand the underlying processes and predict the outcomes of particle interactions.

4. Can Feynman diagrams be used for any type of particle interaction?

Yes, Feynman diagrams can be used to describe any type of particle interaction, including interactions between elementary particles, as well as interactions between particles and anti-particles.

5. Are there any limitations to using Feynman diagrams?

While Feynman diagrams are a powerful tool for understanding particle interactions, they have limitations. They do not take into account the effects of gravity and do not work for extremely high energy interactions. Additionally, they only provide a probability of outcomes rather than a definite prediction.

Similar threads

  • Advanced Physics Homework Help
Replies
6
Views
2K
  • Advanced Physics Homework Help
Replies
1
Views
5K
Replies
1
Views
662
  • Advanced Physics Homework Help
Replies
1
Views
794
  • Advanced Physics Homework Help
Replies
3
Views
1K
  • Advanced Physics Homework Help
Replies
6
Views
1K
  • Advanced Physics Homework Help
Replies
1
Views
2K
  • Advanced Physics Homework Help
Replies
1
Views
2K
  • Advanced Physics Homework Help
Replies
1
Views
3K
  • Advanced Physics Homework Help
Replies
1
Views
2K
Back
Top